If a sequence is not bounded above, there exists an subsequnce that convergest to + infinity











up vote
1
down vote

favorite












i have no idea how to solve that
i have tried by saything that there is a subsequence that always increases but nothing seems to work out










share|cite|improve this question







New contributor




Λάμπρος Αθανασόπουλος is a new contributor to this site. Take care in asking for clarification, commenting, and answering.
Check out our Code of Conduct.




















  • @JoséCarlosSantos They already have two answers from before you posted your comment, but I appreciate that you're saying the question should have shown more effort on their part.
    – Sam Streeter
    Nov 14 at 10:58












  • i am greek so my answers are in greek thats why i ididint post them and im not yet used to english mathematic terms
    – Λάμπρος Αθανασόπουλος
    Nov 14 at 10:58















up vote
1
down vote

favorite












i have no idea how to solve that
i have tried by saything that there is a subsequence that always increases but nothing seems to work out










share|cite|improve this question







New contributor




Λάμπρος Αθανασόπουλος is a new contributor to this site. Take care in asking for clarification, commenting, and answering.
Check out our Code of Conduct.




















  • @JoséCarlosSantos They already have two answers from before you posted your comment, but I appreciate that you're saying the question should have shown more effort on their part.
    – Sam Streeter
    Nov 14 at 10:58












  • i am greek so my answers are in greek thats why i ididint post them and im not yet used to english mathematic terms
    – Λάμπρος Αθανασόπουλος
    Nov 14 at 10:58













up vote
1
down vote

favorite









up vote
1
down vote

favorite











i have no idea how to solve that
i have tried by saything that there is a subsequence that always increases but nothing seems to work out










share|cite|improve this question







New contributor




Λάμπρος Αθανασόπουλος is a new contributor to this site. Take care in asking for clarification, commenting, and answering.
Check out our Code of Conduct.











i have no idea how to solve that
i have tried by saything that there is a subsequence that always increases but nothing seems to work out







analysis






share|cite|improve this question







New contributor




Λάμπρος Αθανασόπουλος is a new contributor to this site. Take care in asking for clarification, commenting, and answering.
Check out our Code of Conduct.











share|cite|improve this question







New contributor




Λάμπρος Αθανασόπουλος is a new contributor to this site. Take care in asking for clarification, commenting, and answering.
Check out our Code of Conduct.









share|cite|improve this question




share|cite|improve this question






New contributor




Λάμπρος Αθανασόπουλος is a new contributor to this site. Take care in asking for clarification, commenting, and answering.
Check out our Code of Conduct.









asked Nov 14 at 10:38









Λάμπρος Αθανασόπουλος

61




61




New contributor




Λάμπρος Αθανασόπουλος is a new contributor to this site. Take care in asking for clarification, commenting, and answering.
Check out our Code of Conduct.





New contributor





Λάμπρος Αθανασόπουλος is a new contributor to this site. Take care in asking for clarification, commenting, and answering.
Check out our Code of Conduct.






Λάμπρος Αθανασόπουλος is a new contributor to this site. Take care in asking for clarification, commenting, and answering.
Check out our Code of Conduct.












  • @JoséCarlosSantos They already have two answers from before you posted your comment, but I appreciate that you're saying the question should have shown more effort on their part.
    – Sam Streeter
    Nov 14 at 10:58












  • i am greek so my answers are in greek thats why i ididint post them and im not yet used to english mathematic terms
    – Λάμπρος Αθανασόπουλος
    Nov 14 at 10:58


















  • @JoséCarlosSantos They already have two answers from before you posted your comment, but I appreciate that you're saying the question should have shown more effort on their part.
    – Sam Streeter
    Nov 14 at 10:58












  • i am greek so my answers are in greek thats why i ididint post them and im not yet used to english mathematic terms
    – Λάμπρος Αθανασόπουλος
    Nov 14 at 10:58
















@JoséCarlosSantos They already have two answers from before you posted your comment, but I appreciate that you're saying the question should have shown more effort on their part.
– Sam Streeter
Nov 14 at 10:58






@JoséCarlosSantos They already have two answers from before you posted your comment, but I appreciate that you're saying the question should have shown more effort on their part.
– Sam Streeter
Nov 14 at 10:58














i am greek so my answers are in greek thats why i ididint post them and im not yet used to english mathematic terms
– Λάμπρος Αθανασόπουλος
Nov 14 at 10:58




i am greek so my answers are in greek thats why i ididint post them and im not yet used to english mathematic terms
– Λάμπρος Αθανασόπουλος
Nov 14 at 10:58










2 Answers
2






active

oldest

votes

















up vote
6
down vote













Choose $n_1$ such that $a_{n_1} >1$ then $n_2>n_1$ such that $a_{n_2} >2$ and so on.






share|cite|improve this answer























  • and why would that help me, i really cant see the solution...
    – Λάμπρος Αθανασόπουλος
    Nov 14 at 10:41










  • @ΛάμπροςΑθανασόπουλος Well, that constructs a subsequence that converges to $+infty$, as desired (you may want to additionlly enforce $n_2>n_1$ etc)
    – Hagen von Eitzen
    Nov 14 at 10:43












  • @HagenvonEitzen is that a strict mathemattic solution tho?
    – Λάμπρος Αθανασόπουλος
    Nov 14 at 10:45


















up vote
0
down vote













Well, since the sequence is not bounded above, the sequence itself goes to infinity. Every sequence is a subsequence of itself, technically, but I imagine you want a proper subsequence.



Well, the simplest way to do as much would be to take successive "peaks" of the sequence. Let the sequence be $(a_n)$. Then just let $a_{n_1} = a_1$, and then let $a_{n_2} = a_k$, where $k$ is the first $k$ such that $a_k > a_{n_1}$. Then let $a_{n_3} = a_m$, where $m$ is the first $m$ such that $a_m > a_{n_2}$, and so on. Each successive term is guaranteed to exist by definition (otherwise the original sequence would converge to something finite, or negative infinity, if at all).



Since the $a_{n_k}$'s define a monotonically increasing sequence, which is unbounded because $(a_n)$ also is, it must approach $infty$.






share|cite|improve this answer





















  • thanks a lot , that helped me a lot
    – Λάμπρος Αθανασόπουλος
    Nov 14 at 10:53










  • but it can also be for ex. -1/n which dosnt have sup and it converges to 0
    – Λάμπρος Αθανασόπουλος
    Nov 14 at 11:01










  • That sequence is not necessarily strictly monoton increasing.
    – Michael Hoppe
    Nov 14 at 11:21











Your Answer





StackExchange.ifUsing("editor", function () {
return StackExchange.using("mathjaxEditing", function () {
StackExchange.MarkdownEditor.creationCallbacks.add(function (editor, postfix) {
StackExchange.mathjaxEditing.prepareWmdForMathJax(editor, postfix, [["$", "$"], ["\\(","\\)"]]);
});
});
}, "mathjax-editing");

StackExchange.ready(function() {
var channelOptions = {
tags: "".split(" "),
id: "69"
};
initTagRenderer("".split(" "), "".split(" "), channelOptions);

StackExchange.using("externalEditor", function() {
// Have to fire editor after snippets, if snippets enabled
if (StackExchange.settings.snippets.snippetsEnabled) {
StackExchange.using("snippets", function() {
createEditor();
});
}
else {
createEditor();
}
});

function createEditor() {
StackExchange.prepareEditor({
heartbeatType: 'answer',
convertImagesToLinks: true,
noModals: true,
showLowRepImageUploadWarning: true,
reputationToPostImages: 10,
bindNavPrevention: true,
postfix: "",
imageUploader: {
brandingHtml: "Powered by u003ca class="icon-imgur-white" href="https://imgur.com/"u003eu003c/au003e",
contentPolicyHtml: "User contributions licensed under u003ca href="https://creativecommons.org/licenses/by-sa/3.0/"u003ecc by-sa 3.0 with attribution requiredu003c/au003e u003ca href="https://stackoverflow.com/legal/content-policy"u003e(content policy)u003c/au003e",
allowUrls: true
},
noCode: true, onDemand: true,
discardSelector: ".discard-answer"
,immediatelyShowMarkdownHelp:true
});


}
});






Λάμπρος Αθανασόπουλος is a new contributor. Be nice, and check out our Code of Conduct.










 

draft saved


draft discarded


















StackExchange.ready(
function () {
StackExchange.openid.initPostLogin('.new-post-login', 'https%3a%2f%2fmath.stackexchange.com%2fquestions%2f2998111%2fif-a-sequence-is-not-bounded-above-there-exists-an-subsequnce-that-convergest-t%23new-answer', 'question_page');
}
);

Post as a guest















Required, but never shown

























2 Answers
2






active

oldest

votes








2 Answers
2






active

oldest

votes









active

oldest

votes






active

oldest

votes








up vote
6
down vote













Choose $n_1$ such that $a_{n_1} >1$ then $n_2>n_1$ such that $a_{n_2} >2$ and so on.






share|cite|improve this answer























  • and why would that help me, i really cant see the solution...
    – Λάμπρος Αθανασόπουλος
    Nov 14 at 10:41










  • @ΛάμπροςΑθανασόπουλος Well, that constructs a subsequence that converges to $+infty$, as desired (you may want to additionlly enforce $n_2>n_1$ etc)
    – Hagen von Eitzen
    Nov 14 at 10:43












  • @HagenvonEitzen is that a strict mathemattic solution tho?
    – Λάμπρος Αθανασόπουλος
    Nov 14 at 10:45















up vote
6
down vote













Choose $n_1$ such that $a_{n_1} >1$ then $n_2>n_1$ such that $a_{n_2} >2$ and so on.






share|cite|improve this answer























  • and why would that help me, i really cant see the solution...
    – Λάμπρος Αθανασόπουλος
    Nov 14 at 10:41










  • @ΛάμπροςΑθανασόπουλος Well, that constructs a subsequence that converges to $+infty$, as desired (you may want to additionlly enforce $n_2>n_1$ etc)
    – Hagen von Eitzen
    Nov 14 at 10:43












  • @HagenvonEitzen is that a strict mathemattic solution tho?
    – Λάμπρος Αθανασόπουλος
    Nov 14 at 10:45













up vote
6
down vote










up vote
6
down vote









Choose $n_1$ such that $a_{n_1} >1$ then $n_2>n_1$ such that $a_{n_2} >2$ and so on.






share|cite|improve this answer














Choose $n_1$ such that $a_{n_1} >1$ then $n_2>n_1$ such that $a_{n_2} >2$ and so on.







share|cite|improve this answer














share|cite|improve this answer



share|cite|improve this answer








edited Nov 14 at 11:58

























answered Nov 14 at 10:40









Kavi Rama Murthy

40.4k31751




40.4k31751












  • and why would that help me, i really cant see the solution...
    – Λάμπρος Αθανασόπουλος
    Nov 14 at 10:41










  • @ΛάμπροςΑθανασόπουλος Well, that constructs a subsequence that converges to $+infty$, as desired (you may want to additionlly enforce $n_2>n_1$ etc)
    – Hagen von Eitzen
    Nov 14 at 10:43












  • @HagenvonEitzen is that a strict mathemattic solution tho?
    – Λάμπρος Αθανασόπουλος
    Nov 14 at 10:45


















  • and why would that help me, i really cant see the solution...
    – Λάμπρος Αθανασόπουλος
    Nov 14 at 10:41










  • @ΛάμπροςΑθανασόπουλος Well, that constructs a subsequence that converges to $+infty$, as desired (you may want to additionlly enforce $n_2>n_1$ etc)
    – Hagen von Eitzen
    Nov 14 at 10:43












  • @HagenvonEitzen is that a strict mathemattic solution tho?
    – Λάμπρος Αθανασόπουλος
    Nov 14 at 10:45
















and why would that help me, i really cant see the solution...
– Λάμπρος Αθανασόπουλος
Nov 14 at 10:41




and why would that help me, i really cant see the solution...
– Λάμπρος Αθανασόπουλος
Nov 14 at 10:41












@ΛάμπροςΑθανασόπουλος Well, that constructs a subsequence that converges to $+infty$, as desired (you may want to additionlly enforce $n_2>n_1$ etc)
– Hagen von Eitzen
Nov 14 at 10:43






@ΛάμπροςΑθανασόπουλος Well, that constructs a subsequence that converges to $+infty$, as desired (you may want to additionlly enforce $n_2>n_1$ etc)
– Hagen von Eitzen
Nov 14 at 10:43














@HagenvonEitzen is that a strict mathemattic solution tho?
– Λάμπρος Αθανασόπουλος
Nov 14 at 10:45




@HagenvonEitzen is that a strict mathemattic solution tho?
– Λάμπρος Αθανασόπουλος
Nov 14 at 10:45










up vote
0
down vote













Well, since the sequence is not bounded above, the sequence itself goes to infinity. Every sequence is a subsequence of itself, technically, but I imagine you want a proper subsequence.



Well, the simplest way to do as much would be to take successive "peaks" of the sequence. Let the sequence be $(a_n)$. Then just let $a_{n_1} = a_1$, and then let $a_{n_2} = a_k$, where $k$ is the first $k$ such that $a_k > a_{n_1}$. Then let $a_{n_3} = a_m$, where $m$ is the first $m$ such that $a_m > a_{n_2}$, and so on. Each successive term is guaranteed to exist by definition (otherwise the original sequence would converge to something finite, or negative infinity, if at all).



Since the $a_{n_k}$'s define a monotonically increasing sequence, which is unbounded because $(a_n)$ also is, it must approach $infty$.






share|cite|improve this answer





















  • thanks a lot , that helped me a lot
    – Λάμπρος Αθανασόπουλος
    Nov 14 at 10:53










  • but it can also be for ex. -1/n which dosnt have sup and it converges to 0
    – Λάμπρος Αθανασόπουλος
    Nov 14 at 11:01










  • That sequence is not necessarily strictly monoton increasing.
    – Michael Hoppe
    Nov 14 at 11:21















up vote
0
down vote













Well, since the sequence is not bounded above, the sequence itself goes to infinity. Every sequence is a subsequence of itself, technically, but I imagine you want a proper subsequence.



Well, the simplest way to do as much would be to take successive "peaks" of the sequence. Let the sequence be $(a_n)$. Then just let $a_{n_1} = a_1$, and then let $a_{n_2} = a_k$, where $k$ is the first $k$ such that $a_k > a_{n_1}$. Then let $a_{n_3} = a_m$, where $m$ is the first $m$ such that $a_m > a_{n_2}$, and so on. Each successive term is guaranteed to exist by definition (otherwise the original sequence would converge to something finite, or negative infinity, if at all).



Since the $a_{n_k}$'s define a monotonically increasing sequence, which is unbounded because $(a_n)$ also is, it must approach $infty$.






share|cite|improve this answer





















  • thanks a lot , that helped me a lot
    – Λάμπρος Αθανασόπουλος
    Nov 14 at 10:53










  • but it can also be for ex. -1/n which dosnt have sup and it converges to 0
    – Λάμπρος Αθανασόπουλος
    Nov 14 at 11:01










  • That sequence is not necessarily strictly monoton increasing.
    – Michael Hoppe
    Nov 14 at 11:21













up vote
0
down vote










up vote
0
down vote









Well, since the sequence is not bounded above, the sequence itself goes to infinity. Every sequence is a subsequence of itself, technically, but I imagine you want a proper subsequence.



Well, the simplest way to do as much would be to take successive "peaks" of the sequence. Let the sequence be $(a_n)$. Then just let $a_{n_1} = a_1$, and then let $a_{n_2} = a_k$, where $k$ is the first $k$ such that $a_k > a_{n_1}$. Then let $a_{n_3} = a_m$, where $m$ is the first $m$ such that $a_m > a_{n_2}$, and so on. Each successive term is guaranteed to exist by definition (otherwise the original sequence would converge to something finite, or negative infinity, if at all).



Since the $a_{n_k}$'s define a monotonically increasing sequence, which is unbounded because $(a_n)$ also is, it must approach $infty$.






share|cite|improve this answer












Well, since the sequence is not bounded above, the sequence itself goes to infinity. Every sequence is a subsequence of itself, technically, but I imagine you want a proper subsequence.



Well, the simplest way to do as much would be to take successive "peaks" of the sequence. Let the sequence be $(a_n)$. Then just let $a_{n_1} = a_1$, and then let $a_{n_2} = a_k$, where $k$ is the first $k$ such that $a_k > a_{n_1}$. Then let $a_{n_3} = a_m$, where $m$ is the first $m$ such that $a_m > a_{n_2}$, and so on. Each successive term is guaranteed to exist by definition (otherwise the original sequence would converge to something finite, or negative infinity, if at all).



Since the $a_{n_k}$'s define a monotonically increasing sequence, which is unbounded because $(a_n)$ also is, it must approach $infty$.







share|cite|improve this answer












share|cite|improve this answer



share|cite|improve this answer










answered Nov 14 at 10:49









Eevee Trainer

95111




95111












  • thanks a lot , that helped me a lot
    – Λάμπρος Αθανασόπουλος
    Nov 14 at 10:53










  • but it can also be for ex. -1/n which dosnt have sup and it converges to 0
    – Λάμπρος Αθανασόπουλος
    Nov 14 at 11:01










  • That sequence is not necessarily strictly monoton increasing.
    – Michael Hoppe
    Nov 14 at 11:21


















  • thanks a lot , that helped me a lot
    – Λάμπρος Αθανασόπουλος
    Nov 14 at 10:53










  • but it can also be for ex. -1/n which dosnt have sup and it converges to 0
    – Λάμπρος Αθανασόπουλος
    Nov 14 at 11:01










  • That sequence is not necessarily strictly monoton increasing.
    – Michael Hoppe
    Nov 14 at 11:21
















thanks a lot , that helped me a lot
– Λάμπρος Αθανασόπουλος
Nov 14 at 10:53




thanks a lot , that helped me a lot
– Λάμπρος Αθανασόπουλος
Nov 14 at 10:53












but it can also be for ex. -1/n which dosnt have sup and it converges to 0
– Λάμπρος Αθανασόπουλος
Nov 14 at 11:01




but it can also be for ex. -1/n which dosnt have sup and it converges to 0
– Λάμπρος Αθανασόπουλος
Nov 14 at 11:01












That sequence is not necessarily strictly monoton increasing.
– Michael Hoppe
Nov 14 at 11:21




That sequence is not necessarily strictly monoton increasing.
– Michael Hoppe
Nov 14 at 11:21










Λάμπρος Αθανασόπουλος is a new contributor. Be nice, and check out our Code of Conduct.










 

draft saved


draft discarded


















Λάμπρος Αθανασόπουλος is a new contributor. Be nice, and check out our Code of Conduct.













Λάμπρος Αθανασόπουλος is a new contributor. Be nice, and check out our Code of Conduct.












Λάμπρος Αθανασόπουλος is a new contributor. Be nice, and check out our Code of Conduct.















 


draft saved


draft discarded














StackExchange.ready(
function () {
StackExchange.openid.initPostLogin('.new-post-login', 'https%3a%2f%2fmath.stackexchange.com%2fquestions%2f2998111%2fif-a-sequence-is-not-bounded-above-there-exists-an-subsequnce-that-convergest-t%23new-answer', 'question_page');
}
);

Post as a guest















Required, but never shown





















































Required, but never shown














Required, but never shown












Required, but never shown







Required, but never shown

































Required, but never shown














Required, but never shown












Required, but never shown







Required, but never shown







Popular posts from this blog

Plaza Victoria

In PowerPoint, is there a keyboard shortcut for bulleted / numbered list?

How to put 3 figures in Latex with 2 figures side by side and 1 below these side by side images but in...